2014 AMC 12A
by djmathman, Feb 5, 2014, 1:16 AM
I took it. It was like an AMC.
On a side note, I have all the problems typed up and ready to post on the fora tomorrow!
(darn some Asymptote diagrams can be soooo frustrating to code)
On another side note, here's some math I've done recently that has nothing to do with AMC's.
Solution
Solution
On a side note, I have all the problems typed up and ready to post on the fora tomorrow!

On another side note, here's some math I've done recently that has nothing to do with AMC's.
India 2014.1 wrote:
In a triangle
, let
be the point on the segment
such that
. Suppose that the points
,
and the centroids of triangles
and
lie on a circle. Prove that
.









Solution
Let
be the centroids of
,
respectively. Note that as
, we have
. Since
is a cyclic quadrilateral, it must be isosceles, so
. As
and
concur at the midpoint
of
, it must hold true that
.
Now from either the Appolonius Theorem for Medians or some Law of Cosines (if you're one of those people like me who keeps forgetting the formula), it is true that
It is given that
, so through some manipulation
. Thus the roots of
and
concur, whence
. However, if
and
, then
, contradiction. Therefore
as desired. 












Now from either the Appolonius Theorem for Medians or some Law of Cosines (if you're one of those people like me who keeps forgetting the formula), it is true that
\[\begin{align*}\dfrac{2AB^2+2BD^2-AD^2}4=BG^2&=CG^2=\dfrac{2AC^2+2CD^2-AD^2}4\\\implies AB^2+BD^2&=AC^2+CD^2.\end{align*}\]
It is given that










India 2014.5 wrote:
In a acute-angled triangle
, a point
lies on the segment
. Let
denote the circumcentres of triangles
and
respectively. Prove that the line joining the circumcentre of triangle
and the orthocentre of triangle
is parallel to
.









Solution
Let
be the circumcenter of
and
its orthocenter. Furthermore, let
be the midpoint of
and
the foot of the altitude from
to
. Note that from some simple angle chasing we get
, so considering the spiral similarity sending the former triangle to the latter gives that
. As
is the perpendicular bisector of
,
, so
, the orthocenter of
, is the reflection of
across
. Thus, quadrilaterals
and
are similar. Now, let
be the distance from
to the line
, and note that
![\[\dfrac d{AX}=\dfrac{H_1D}{AD}=\dfrac{H_1D}{2AM}=\dfrac{AH}{2AX}\implies d=\tfrac12AH.\]](//latex.artofproblemsolving.com/b/9/7/b97d1df994e923389bdc07aeac35225e9e9dd7c4.png)
Thus the distance is fixed, and so
lies on a line parallel to
. All that remains is to show that
lies on this line, but this is easy, as it is well known that
and that the distance from
to
is
, where
is the circumradius of
. 






















![\[\dfrac d{AX}=\dfrac{H_1D}{AD}=\dfrac{H_1D}{2AM}=\dfrac{AH}{2AX}\implies d=\tfrac12AH.\]](http://latex.artofproblemsolving.com/b/9/7/b97d1df994e923389bdc07aeac35225e9e9dd7c4.png)
Thus the distance is fixed, and so









